Discussion

Which one of the following, if true, most seriously weakens the argument?
(A)Of the respondents who had received treatment for longer than 6 months, 10 percent said that treatment made things worse.
(B)...
(C)...
(D)...
(E)...
(F)...
*This question is included in Free Complete Section: LR-B, June '07 LSAT, question #15

The solution is

Posted: 01/25/2012 09:13
Why is answer C correct?
Image Not Available
Contributor
Posted: 01/25/2012 14:23
Tyrone, the conclusion is based upon the assumption that "How people feel" depends on the time they have been treated, while answer C is claiming the exact opposite; "How people feel" will determine the length of treatment and it will make the conclusion proposed(even the whole survey) meaningless :)

Niels
Posted: 11/17/2012 23:33
Why can't B be the correct answer?
Image Not Available
Contributor
Posted: 11/18/2012 01:04
Keti,

(B) Patients who had received treatment for longer than 6 months were more likely to respond to the survey than were those who had received treatment for a shorter time.

Ask yourselves: If this would be true, what would be the outcome?
Well, the influence against the percentage of ALL the people who responded is NONE
There is however influence in the percentage of responders from a certain group the 'longer than six months'-people.
Now the argument states: Longer treatment yields better results.
While, if true there are more responders from a particular group it doesn't imply the returned answers are influenced.
Thus the frequency of responders has no influence on the answer replied.

Niels

PS.
For clarity, if there would have been 50 short responders saying: Splendid! And there are 5000 long responders saying: Yak! The outcome of the survey would be that short treatment is better while B could be true.
Posted: 11/18/2012 20:35
Thanks Niels.
Image Not Available
Contributor
Posted: 11/20/2012 07:00
You're welcome :)
Posted: 07/06/2013 11:51
I still don't understand why "C" is the answer
Posted: 07/08/2013 14:13
Tolu, Niels put it succinctly in the prior post, that the problem makes the claim that efficacy of treatment corresponds to the length of treatment time. Answer C makes this claim invalid, since it does not take into account that people whose treatment is not effective will quit before 6 months.

Here is a link to a video explanation by a professional LSAT tutor: http://www.youtube.com/watch?v=hb7mou0hDJU
Posted: 07/10/2013 21:18
Hearing him breaking it down helped a lot. Thanks!
Posted: 02/06/2015 05:18
I was torn between A & C and my gut said C but I hit A. The manner in which this weakens the argument is very obtuse to discern but this explanation helps test takers watch out for these sneaky, somewhat hidden direct connections to the conclusion
Posted: 05/22/2015 13:49
So true.
Posted: 05/22/2015 15:21
So do you work for LSAC? Or is this your own gig you have going here?
| Edit

You need to be signed in to perform that action.

Sign In